0
$\begingroup$

Given $m \ge 0$ and $n$ odd, $ s >0 $ is there a way, for computing asymptotic expansion of following integral: $$ \int_{\mathcal{C}} \int_{\mathcal{C}} \frac{ \Gamma(m + s_1) \Gamma(m + s_2) \Gamma(1 - \frac{n}{2} + s_1) \Gamma(1 - \frac{n}{2} + s_2) \Gamma(-s_1) \Gamma(-s_2) \Gamma(m + \frac{n}{2} + s_1 + s_2) }{ \Gamma(m + \frac{n}{2} + s_1) \Gamma(m + \frac{n}{2} + s_2) \Gamma(m + \frac{n}{2} + s_1 + s_2 + s + 1)}\;ds_1\,ds_2\,, $$ where $C$ is contour avoiding all singularities? I tried to sum up all possible combinations of residues arising from $s_1$ and $s_2$, unfortunately this does not lead to anything useful. Is there at least way to show, that residues emerging from $(s_1,s_2) \in \mathbb{N_0}\times\mathbb{N_0}$ are dominant in total sum? Thanks.

$\endgroup$
1
  • $\begingroup$ This contour $C$ starts where and ends where? Asymptotic expansion in what? $\endgroup$ Commented May 13 at 13:14

0

You must log in to answer this question.

Start asking to get answers

Find the answer to your question by asking.

Ask question

Explore related questions

See similar questions with these tags.